Gibt es noch Lagrange-Punkte, wenn vom ersten auf den dritten Körper ein erheblicher Strahlungsdruck ausgeübt wird?

Lagrange-Punkte-Diagramm

Aus dieser Antwort :

Um den Abstand zu L1 zu erhalten, suchen Sie den kleinsten Wert von R so dass

M 2 R 1 2 + M 1 R 2 R 1 ( M 1 + M 2 ) R 3 M 1 ( R R 1 ) 2 = 0.

Um den Abstand zu L2 zu erhalten, suchen Sie den kleinsten Wert von R so dass

M 1 R 2 + R 2 ( M 1 + M 2 ) R 3 M 1 ( R + R 2 ) 2 M 2 R 2 2 = 0.

Oben erfahren Sie, wie Sie Entfernungen berechnen M 2 zum L 1 Und L 2 Punkte. Diese Lösungen repräsentieren das Gleichgewicht zwischen Gravitations- und Zentripetalkräften im mitrotierenden Rahmen.

Nehmen wir nun an, dass der dritte Körper eine reduzierte umgekehrte quadratische Kraft erfährt M 1 um einen Faktor δ was der Fall sein könnte, wenn es den Strahlungsdruck der Sonne spürte. Die Kraft aus M 2 würde aber unverändert aus M 1 es würde um einen Faktor skaliert werden 1 δ .

Fragen:

  1. Kann gezeigt werden (anstatt nur zu sagen), dass die Lagrange-Punkte immer noch existieren und sich genauso verhalten, sich aber an einem anderen Ort befinden?
  2. Wenn ja, welche Gleichung müsste für das Neue gelöst werden? R 1 Und R 2 für ein gegebenes δ ?

"Bonuspunkte:" Kann gezeigt werden (anstatt nur zu sagen), dass Halo-Umlaufbahnen immer noch existieren und sich für Nicht-Null auf ähnliche Weise verhalten würden δ ?

Lagrange-Punkte sind eine mathematische Abstraktion, die zufällig Punkten im Raum entsprechen, die bestimmte nützliche Gravitationseigenschaften haben. Aber diese Orte im Weltraum sind möglicherweise aus einer Reihe anderer Gründe praktisch nicht nutzbar. Einige dieser Gründe werden dazu führen, dass sich ein anderer Ort im Weltraum so verhält, wie es der Lagrange-Punkt tun sollte. Aber dieser Ort ist nicht wirklich der Lagrange-Punkt. Die Lagrange-Punkte existieren immer, auch wenn sie zufällig unter der Planetenoberfläche liegen.
@SF. also nennen wir sie bei Bedarf Uhoh-Punkte, aber ich glaube wirklich nicht, dass Lagrange etwas gegen eine kleine Änderung der Kräfte haben würde, die in einer ansonsten identischen Ableitung verwendet werden.
Nennen Sie sie Kraftgleichgewichtspunkte und ich glaube nicht, dass es irgendjemandem etwas ausmachen wird.
@SF. vielleicht können wir sie noch Librationspunkte nennen, da ich nur nach 1 und 2 gefragt habe, so können wir das "L" behalten
Eh, das ist Stackexchange, also habe ich beide bearbeitet. Werfen Sie einen zweiten Blick und machen Sie einen Rollback, wenn ich völlig verrückt bin.
@Hohmannfan danke sowohl für die Antwort als auch für die Abhilfemaßnahmen!

Antworten (1)

1) und 2) sind einfach zu zeigen, der Bonus ist sehr hart und ich werde es nicht versuchen.

A L Der Überlagerungspunkt kann als Gleichgewicht zwischen drei Beschleunigungen in einem rotierenden Bezugssystem angesehen werden.

  1. Schwerkraft aus M 1
  2. Schwerkraft aus M 2
  3. Zentrifugalbeschleunigung.

Für L 2 , die ersten beiden sind ( 1 δ ) M 1 ( R + R 2 ) 2 Und M 2 R " 2 bzw. Dein δ inbegriffen.

Die dritte Beschleunigung wäre ω 2 R C e N T R e , Wo ω 2 = μ R 3 Und R C e N T R e = R M 1 μ + R 2

Wir haben dann:

M 2 R 2 2 ( 1 δ ) M 1 ( R + R 2 ) 2 + M 1 + M 2 R 3 ( R 2 + R M 1 M 1 + M 2 ) = 0

Was vereinfacht zu:

M 1 R 2 + R 2 ( M 1 + M 2 ) R 3 ( 1 δ ) M 1 ( R + R 2 ) 2 M 2 R 2 2 = 0

Was Ihrer zweiten Formel unverkennbar ähnelt.

Der Vollständigkeit halber hier L 1 :

M 1 R 2 + R 1 ( M 1 + M 2 ) R 3 ( 1 δ ) M 1 ( R + R 1 ) 2 + M 2 R 1 2 = 0

Und L 3 :

M 1 R 2 R 3 ( M 1 + M 2 ) R 3 + ( 1 δ ) M 1 ( R + R 3 ) 2 + M 2 R 3 2 = 0

Diese Herleitung sollte 2) beantworten. Aber existiert es?

Dafür kann ein wesentlich einfacheres Argument verwendet werden.

Angenommen, wir ziehen um L 2 nach innen zum zweiten Körper:

  1. Die Schwerkraft aus M 1 wächst, aber nur auf den festen Wert im Abstand des zweiten Körpers zu.
  2. Die Schwerkraft aus M 2 wächst, und es tendiert schnell ins Unendliche als die L 2 nähert sich dem Massepunkt
  3. Die Zentrifugalbeschleunigung nimmt ab.

Daraus folgt, dass jede Erhöhung der Beschleunigung aus M 1 kann mit dem beliebig hohen Wert für die Kombination der beiden anderen Beschleunigungen begegnet werden.

Dasselbe Argument kann für das Wegbewegen vom zweiten Körper angeführt werden . Die Zentrifugalbeschleunigung wächst linear beliebig hoch, während die entgegenwirkende Schwerkraft mit dem Quadrat der Entfernung schrumpft, bis die Gleichung im Gleichgewicht ist.

L 2 existiert immer

Dasselbe gilt jedoch nicht für L 1 . Während eine Erhöhung der Beschleunigung aus M 1 kann durch Bewegung entgegengewirkt werden L 1 willkürlich schließen das der zweite Körper, eine Beschleunigungsabnahme darüber hinaus 1 δ = 0 führt dazu, dass alle Beschleunigungen in die gleiche Richtung gehen. Tatsächlich müsste man sich in diesem Fall auf der gegenüberliegenden Seite des zentralen Körpers befinden L 2 L 3

Wow, danke für die schnelle und dennoch gründliche Antwort!
@uhoh Ich denke, Sie werden diese Handlung mögen, in der ein Sonnensegel in einem Zwei-Körper-System schweben kann: i.stack.imgur.com/ECF0V.png
Stabile Sonnensegelregionen mit Delta 0 bis 0,1: i.stack.imgur.com/HbPX3.png
Eine Animation: i.fiery.me/sWMiL.webp
Ich brauche etwas Kaffee, um diese zu verarbeiten; Ich hatte Pläne für heute, jetzt sieht es so aus, als würde ich das tun, vielen Dank! ;-)
Ich habe etwas Kaffee getrunken und einige Tage sind vergangen, und ich kann immer noch nicht erraten, warum dieses Bild eine so hohe Variationsdichte in Delta hat. Ich habe noch nicht versucht, es zu berechnen, aber es macht einfach keinen Sinn für mich. Können Sie mir sagen, was „stabile Sonnensegelregion“ in diesem Zusammenhang genau bedeutet? Meinen Sie damit, dass das Objekt im rotierenden Rahmen fixiert bleibt oder nur, dass es dort eine geschlossene, periodische Umlaufbahn haben kann? Zeigt das Segel immer direkt auf die Sonne oder bleibt es in einem festen Sonnenwinkel oder ändert es sich im Laufe der Zeit?
@uhoh Alle sind dafür da, im rotierenden Rahmen fixiert zu bleiben. Die Bereiche mit sehr hohen Delta-Variationen werden durch den sehr schlechten Arbeitswinkel verursacht, der erforderlich ist, um an Ort und Stelle zu bleiben. Der erforderliche Beschleunigungsvektor steht aufgrund der Schwerkraft des Planeten fast senkrecht zur Sonne. Daher zeigt das Sonnensegel mit seiner Kante fast zur Sonne und erhält nur sehr wenig Beschleunigung.
(nur für den Fall, das Delta schwingt nicht wild, es wächst nur exponentiell über die Grenzen der verwendeten Farbskala hinaus)
ja, du hast meine nächste Frage genau vorweggenommen ;-)
Hallo Leute ... Mir ist klar, dass ich hier überfordert bin ... aber ich würde gerne auf das Konzept des Sonnenstrahlungsdrucks eingehen. Wäre als „Druck“-Kraft, die ausgeübt wird, nicht ein „beeinflusster Oberflächenbereich“ erforderlich? Ein dritter Körper mit hoher Masse, aber einer Oberfläche nahe Null würde sehr wenig beeinflusst werden, während ein Körper mit geringer Masse und großer „voller Frontfläche“ stark beeinflusst würde. Die Berechnung des Sonnendrucks an eine Masse zu binden, macht für mich einfach keinen Sinn.